LSAT and Law School Admissions Forum

Get expert LSAT preparation and law school admissions advice from PowerScore Test Preparation.

 Administrator
PowerScore Staff
  • PowerScore Staff
  • Posts: 8949
  • Joined: Feb 02, 2011
|
#81032
Complete Question Explanation

Flaw in the Reasoning. The correct answer choice is (A).

Answer choice (A): This is the correct answer choice.

Answer choice (B):

Answer choice (C):

Answer choice (D):

Answer choice (E):

This explanation is still in progress. Please post any questions below!
 maximbasu
  • Posts: 59
  • Joined: May 19, 2016
|
#28163
Hi,

I chose C while the correct answer was A.

The stimulus states:
1. there's a link btw cancer + pollutants
2. fish ingest pollutants + people eat fish, thereby get cancer
3. aim: prevent cancer
4. Conclusion: we need to stop all industries producing pollutants, as industries are incompetent + wont do anything if we ask them to stop

Task: ID the flaw

My reasoning for C: If effects, aka cancer, is produced by different causes, stopping all industries will not solve the problem of cancer. There will be other causes, such as stress.
Is C wrong because it doesn't address the pollutants directly + instead states "several different effects?"

A seems to follow the line of reasoning I used for choosing C; what is the difference btw. A and C please?

MB (Official Initials)
 Nikki Siclunov
PowerScore Staff
  • PowerScore Staff
  • Posts: 1362
  • Joined: Aug 02, 2011
|
#28210
Hi MB,

Thanks for your question. Close reading is key - both of the stimulus and of the answer choices.

Your analysis of the stimulus is correct. However, you misread answer choice (C). When the answer to a Flaw question begins with the phrase "takes for granted that...", such an answer choice suggests an unwarranted assumption. Did the author assume that "certain effects can be produced independently by several different causes"? Nope. The author did not assume this possibility; he ignored it. This is indeed the weakness in the argument, and if answer choice (C) had stated this issue as something ignored (or overlooked) by the author, it would have been correct. This is precisely what answer choice (A) does.

Check out this blog post about the interplay between Assumption and Weaken questions in the context of a Flaw in the Reasoning question stem.

Hope this helps! :)
 tamarisk
  • Posts: 4
  • Joined: Sep 15, 2016
|
#28842
Would then answer D be wrong for the simple fact that this answer is a solution to the the premise/sub-conclusion about industries unlikely to comply to strict environmental guidelines? I had answered correctly A, but both A and D seem so plausible since they both delved into the issue at hand in the conclusion sentence. Which, from my experience practicing the prep tests rare and very difficult to decide which answer is the better one.
 Clay Cooper
PowerScore Staff
  • PowerScore Staff
  • Posts: 241
  • Joined: Jul 03, 2015
|
#28855
Hi tamarisk,

Thanks for your question! I think your analysis is pretty astute - both A and D accurately describe what happens in the stimulus, and each could be called a flaw in the author's reasoning.

The question, then, becomes: which is the bigger, more serious flaw? Is it that the activist mentions one plausible cause of cancer and other diseases, and then, on that basis, concludes that only one course of action will work to reduce cancer, etc - totally ignoring other potential causes of the diseases?

Or is it that the author is assuming that industries which currently pollute will not spontaneously decide to self-regulate in the interest of the public health?

I think it is clearly the former - there's no telling what else could cause these diseases, or what fraction of their overall incidence results from pollutants; therefore, concluding that a certain anti-pollution approach is our only hope to combat these diseases is erroneous.

I hope that helps!
 mN2mmvf
  • Posts: 113
  • Joined: Jul 06, 2017
|
#38468
I saw the potential "alternate cause" flaw in the argument, but to me what most stood out as a flaw was the fact that the conclusion argued for was disproportionate to the problem of industries being unlikely to comply adequately with regulations. The activist proposes something pretty radical to address that: halting the industries altogether. I thought (D) seemed like a better choice because it pointed out that the industries might stop polluting even if they didn't "adequately adhere to strict environmental regulations" -- they might do it for another reason, voluntarily.

Where am I going wrong? The alternate cause issue could definitely be a flaw, I agree, but it doesn't seem like that's where the activist's argument is leading at all.
 AthenaDalton
PowerScore Staff
  • PowerScore Staff
  • Posts: 296
  • Joined: May 02, 2017
|
#38624
Hi mN2,

If the activist is correct that companies in this industry would willfully violate strict environmental regulations meant to reduce the number of cancer-causing pollutants their companies produce, it's unlikely that these same companies would adopt voluntary reductions in their activities which contribute to environmental pollution.

In any case, "voluntarily decreasing outputs of pollutants" and "complying with government limits of pollutants" amount to pretty much the same thing -- if a corporation reduces its output of pollutants for moral reasons (e.g., it would have done so even in the absence of a governmental regulation), it can still be in compliance with environmental laws.

The reason (A) is a better answer than (D) is that the activist is saying (1) there's a "link" between water pollutants and cancer / birth defects and (2) the only way to prevent cancer / birth defects is to shut down polluting industries. The activist makes quite a jump from a "link" from pollutants :arrow: cancer and totally eradicating the disease. The activist is assuming that pollutants are the only cause of cancer, when in fact many factors could contribute to cancer: genetics, lifestyle choices, pollutants other than those in the water, exposure to radiation, etc. We just don't have enough information to say with any certainty that eliminating water pollution will eliminate cancer.

I hope this makes sense. Good luck studying!

Athena Dalton
 deck1134
  • Posts: 160
  • Joined: Jun 11, 2018
|
#48962
Hi PowerScore Staff,

Hope all is well.

I am not sure why C is wrong here. I was able to prephrase that "Either there could be an alternate cause (A) or the effects could be produced by several different causes (C)"

Why is A right but C is wrong?

Thank you
 Adam Tyson
PowerScore Staff
  • PowerScore Staff
  • Posts: 5400
  • Joined: Apr 14, 2011
|
#49089
Hey there Deck, did you read Nikki's explanation earlier in this thread? He nailed it perfectly - C is an opposite answer, because the author didn't take that for granted (meaning assumed it was true), but rather failed to consider it (assumed it wasn't true). If the author had assumed the possibility of alternate causes, he wouldn't have been so sure that dealing with this one cause was the only way to solve the problem, right?
 nihals23
  • Posts: 16
  • Joined: Oct 01, 2018
|
#59216
Only for reference purposes, could you give a hypothetical example (or two) for 'preventable factors other than industrial pollutants' mentioned in option A? Thanks :)

Get the most out of your LSAT Prep Plus subscription.

Analyze and track your performance with our Testing and Analytics Package.